Show limit of improper integral is 0

Click For Summary
The discussion centers on whether the limit of the improper integral of a bounded, continuous, non-negative function f converges to zero as x approaches infinity, given that the integral is finite for all x. Participants express an intuitive belief in the truth of this statement and seek to prove it, particularly focusing on the relationship between the limit of f(t) as t approaches infinity and the convergence of the integral. The dominated convergence theorem is suggested as a potential tool for the proof, though its application is debated due to the changing limits of integration. The conversation highlights the need for a rigorous approach to establish the connection between the behavior of f(t) and the convergence of the integral. Ultimately, the discussion emphasizes the importance of mathematical theorems in proving convergence properties of improper integrals.
tjkubo
Messages
41
Reaction score
0

Homework Statement


Suppose f is real-valued, bounded, continuous, and non-negative and suppose \int_x^\infty f(t)\,dt is convergent (is finite) for all x. Is it true that
\lim_{x\rightarrow \infty} {\int_x^\infty f(t)\,dt} = 0 \ ?

Homework Equations



The Attempt at a Solution


I can't think of a counterexample and it seems true intuitively, so I'm trying to prove it's true.
Given \epsilon >0, I want to show there is M such that
\int_M^\infty f(t)\,dt < \epsilon \ .
I think \lim_{t\rightarrow \infty} {f(t)} = 0 although I am not sure how to prove this. Specifically, how would you prove that the improper integral would not exist if this limit did not exist?
I'm not sure what to do at this point.
 
Physics news on Phys.org
tjkubo said:

Homework Statement


Suppose f is real-valued, bounded, continuous, and non-negative and suppose \int_x^\infty f(t)\,dt is convergent (is finite) for all x. Is it true that
\lim_{x\rightarrow \infty} {\int_x^\infty f(t)\,dt} = 0 \ ?

Homework Equations



The Attempt at a Solution


I can't think of a counterexample and it seems true intuitively, so I'm trying to prove it's true.
Given \epsilon >0, I want to show there is M such that
\int_M^\infty f(t)\,dt < \epsilon \ .
I think \lim_{t\rightarrow \infty} {f(t)} = 0 although I am not sure how to prove this. Specifically, how would you prove that the improper integral would not exist if this limit did not exist?
I'm not sure what to do at this point.

Try looking at the dominated convergence theorem.
 
I don't know how that would help since the set we're integrating over is changing with the limit.
 
Ah, so you can use that theorem??

Note that

\int_x^{+\infty}{f(t)dt}=\int_{-\infty}^{+\infty}{I_{[x,+\infty[}(t)f(t)dt}

with

I_{[x,+\infty[}:\mathbb{R}\rightarrow \mathbb{R}:t\rightarrow \left\{\begin{array}{cc} 0 & \text{if}~t<x\\ 1 & \text{if}~t\geq x \end{array}\right.

So what does the dominated convergence theorem tell you?
 
Question: A clock's minute hand has length 4 and its hour hand has length 3. What is the distance between the tips at the moment when it is increasing most rapidly?(Putnam Exam Question) Answer: Making assumption that both the hands moves at constant angular velocities, the answer is ## \sqrt{7} .## But don't you think this assumption is somewhat doubtful and wrong?

Similar threads

  • · Replies 15 ·
Replies
15
Views
2K
Replies
6
Views
2K
Replies
2
Views
2K
  • · Replies 2 ·
Replies
2
Views
2K
  • · Replies 16 ·
Replies
16
Views
2K
  • · Replies 7 ·
Replies
7
Views
2K
  • · Replies 8 ·
Replies
8
Views
2K
  • · Replies 6 ·
Replies
6
Views
2K
Replies
1
Views
2K
  • · Replies 11 ·
Replies
11
Views
2K